Difference between revisions of "2020 AMC 12B Problems/Problem 4"

(Problem)
Line 1: Line 1:
 
==Problem==
 
==Problem==
The acute angles of a right triangle are <math>a^\circ</math> and <math>b^\circ</math>, where <math>a>b</math> and both <math>a</math> and <math>b</math> are prime numbers. What is the least possible value of <math>b</math>?  
+
The acute angles of a right triangle are <math>a^{\circ}</math> and <math>b^{\circ}</math>, where <math>a>b</math> and both <math>a</math> and <math>b</math> are prime numbers. What is the least possible value of <math>b</math>?
  
 +
<math>\textbf{(A) }2\qquad\textbf{(B) }3\qquad\textbf{(C) }5\qquad\textbf{(D) }7\qquad\textbf{(E) }11</math>
  
 
==Solution==
 
==Solution==

Revision as of 21:41, 7 February 2020

Problem

The acute angles of a right triangle are $a^{\circ}$ and $b^{\circ}$, where $a>b$ and both $a$ and $b$ are prime numbers. What is the least possible value of $b$?

$\textbf{(A) }2\qquad\textbf{(B) }3\qquad\textbf{(C) }5\qquad\textbf{(D) }7\qquad\textbf{(E) }11$

Solution

$a+b+90=180$, so $a+b=90$. The largest primes less than $90$ are $89, 83, 79, ...$ If $a=89$, then $b=1$, which is not prime. However, if $a=83$, then $b=7$, which is prime. Hence the answer is $\boxed{\mathrm{(D)}}$

See Also

2020 AMC 12B (ProblemsAnswer KeyResources)
Preceded by
Problem 3
Followed by
Problem 5
1 2 3 4 5 6 7 8 9 10 11 12 13 14 15 16 17 18 19 20 21 22 23 24 25
All AMC 12 Problems and Solutions

The problems on this page are copyrighted by the Mathematical Association of America's American Mathematics Competitions. AMC logo.png